Newline fixes
This commit is contained in:
parent
24fadf347a
commit
6a4645b72a
@ -24,7 +24,7 @@
|
||||
|
||||
|
||||
\begin{solution}
|
||||
In order to completely cover an equilateral triangle, the two smaller triangles must cover all three vertices. Since the longest length of an equilateral triangle is one of its sides, a smaller triangle cannot cover more than one vertex. Therefore, we cannot completely cover the triangle with two smaller copies. \\
|
||||
In order to completely cover an equilateral triangle, the two smaller triangles must cover all three vertices. Since the longest length of an equilateral triangle is one of its sides, a smaller triangle cannot cover more than one vertex. Therefore, we cannot completely cover the triangle with two smaller copies. \par
|
||||
|
||||
\textcolor{gray}{\textit{Bonus question:}} Can you cover a square with three smaller squares?
|
||||
\end{solution}
|
||||
@ -36,15 +36,15 @@
|
||||
|
||||
\problem{}
|
||||
\note{Difficulty: Hard}
|
||||
You are given $n + 1$ integers. \\
|
||||
You are given $n + 1$ integers. \par
|
||||
Prove that at least two of them have a difference divisible by $n$.
|
||||
|
||||
\begin{solution}
|
||||
$n~|~(a-b) \iff a \equiv b \pmod{n}$ \\
|
||||
$n~|~(a-b) \iff a \equiv b \pmod{n}$ \par
|
||||
|
||||
Let $i_0 ... i_{n+1}$ be our set of integers. If we pick $i_0 ... i_{n+1}$ so that no two have a difference divisible by $n$, we must have $i_0 \not\equiv i_k \pmod{n}$ for all $1 \leq k \leq n+1$. There are $n$ such $i_k$, and there are $n$ equivalence classes mod $n$. \\
|
||||
Let $i_0 ... i_{n+1}$ be our set of integers. If we pick $i_0 ... i_{n+1}$ so that no two have a difference divisible by $n$, we must have $i_0 \not\equiv i_k \pmod{n}$ for all $1 \leq k \leq n+1$. There are $n$ such $i_k$, and there are $n$ equivalence classes mod $n$. \par
|
||||
|
||||
Therefore, either, $i_1 ... i_{n+1}$ must cover all equivalence classes mod $n$ (implying that $i_0 \equiv i_k \pmod{n}$ for some k), or there exist two elements in $i_1 ... i_{n+1}$ that are equivalent mod $n$. \\
|
||||
Therefore, either, $i_1 ... i_{n+1}$ must cover all equivalence classes mod $n$ (implying that $i_0 \equiv i_k \pmod{n}$ for some k), or there exist two elements in $i_1 ... i_{n+1}$ that are equivalent mod $n$. \par
|
||||
|
||||
In either case, we can find $a, b$ so that $a \equiv b \pmod{n}$, which implies that $n$ divides $a-b$.
|
||||
\end{solution}
|
||||
@ -60,9 +60,9 @@
|
||||
You have an $8 \times 8$ chess board with two opposing corner squares cut off. You also have a set of dominoes, each of which is the size of two squares. Is it possible to completely cover the the board with dominos, so that none overlap nor stick out?
|
||||
|
||||
\begin{solution}
|
||||
A domino covers two adjacent squares. Adjacent squares have different colors. \\
|
||||
A domino covers two adjacent squares. Adjacent squares have different colors. \par
|
||||
|
||||
If you remove two opposing corners of a chessboard, you remove two squares of the same color, and you're left with $32$ of one and $30$ of the other. \\
|
||||
If you remove two opposing corners of a chessboard, you remove two squares of the same color, and you're left with $32$ of one and $30$ of the other. \par
|
||||
|
||||
Since each domino must cover two colors, you cannot cover the modified board.
|
||||
\end{solution}
|
||||
@ -76,7 +76,7 @@
|
||||
The ocean covers more than a half of the Earth's surface. Prove that the ocean has at least one pair of antipodal points.
|
||||
|
||||
\begin{solution}
|
||||
Let $W$ be the set of wet points, and $W^c$ the set of points antipodal to those in $W$. $W$ and $W^c$ each contain more than half of the points on the earth. The set of dry points, $D$, contains less than half of the points on the earth. Therefore, $W^c \not\subseteq D$. \\
|
||||
Let $W$ be the set of wet points, and $W^c$ the set of points antipodal to those in $W$. $W$ and $W^c$ each contain more than half of the points on the earth. The set of dry points, $D$, contains less than half of the points on the earth. Therefore, $W^c \not\subseteq D$. \par
|
||||
|
||||
\textcolor{gray}{\textit{Note:}} This solution isn't very convincing. However, it is unlikely that the students know enough to provide a fully rigorous proof.
|
||||
\end{solution}
|
||||
@ -90,9 +90,9 @@
|
||||
There are $n > 1$ people at a party. Prove that among them there are at least two people who have the same number of acquaintances at the gathering. (We assume that if A knows B, then B also knows A)
|
||||
|
||||
\begin{solution}
|
||||
Assume that every attendee knows a different number of people. There is only one way this may happen: the most popular person knows $n-1$ people (that is, everyone but himself), the second-most popular knows $n-2$, etc. The least-popular person must then know $0$ people. \\
|
||||
Assume that every attendee knows a different number of people. There is only one way this may happen: the most popular person knows $n-1$ people (that is, everyone but himself), the second-most popular knows $n-2$, etc. The least-popular person must then know $0$ people. \par
|
||||
|
||||
This is impossible, since we know that someone must know $n-1$. \\
|
||||
This is impossible, since we know that someone must know $n-1$. \par
|
||||
(Remember, ``knowing'' must be mutual.)
|
||||
\end{solution}
|
||||
|
||||
@ -106,13 +106,13 @@
|
||||
Pick five points in $\mathbb{R}^2$ with integral coordinates. Show that two of these form a line segment that has an integral midpoint.
|
||||
|
||||
\begin{solution}
|
||||
Let $e, o$ represent even and odd integers. \\
|
||||
There are four possible classes of points: $(e, e)$, $(o, o)$, $(e, o)$, $(o, e)$. \\
|
||||
Let $e, o$ represent even and odd integers. \par
|
||||
There are four possible classes of points: $(e, e)$, $(o, o)$, $(e, o)$, $(o, e)$. \par
|
||||
|
||||
$\text{midpoint}(a, b) = (\frac{a_x + b_x}{2}, \frac{a_y + b_y}{2})$. If $a_x + b_x$ and $a_y + b_y$ are both even, the midpoint of points $a$ and $b$ will have integer coordinates. \\
|
||||
$\text{midpoint}(a, b) = (\frac{a_x + b_x}{2}, \frac{a_y + b_y}{2})$. If $a_x + b_x$ and $a_y + b_y$ are both even, the midpoint of points $a$ and $b$ will have integer coordinates. \par
|
||||
|
||||
Since we pick five points from four classes, at least two must come from the same class. \\
|
||||
$e + e = e$ and $o + o = e$, so the midpoint between two points of the same class must have integral coordinates. \\
|
||||
Since we pick five points from four classes, at least two must come from the same class. \par
|
||||
$e + e = e$ and $o + o = e$, so the midpoint between two points of the same class must have integral coordinates. \par
|
||||
|
||||
\end{solution}
|
||||
|
||||
@ -126,7 +126,7 @@
|
||||
Every point on a line is painted black or white. Show that there exist three points of the same color where one is the midpoint of the line segment formed by the other two.
|
||||
|
||||
\begin{solution}
|
||||
This is a proof by contradiction. We will try to construct a set of points where three points have such an arrangement. \\
|
||||
This is a proof by contradiction. We will try to construct a set of points where three points have such an arrangement. \par
|
||||
|
||||
We know that some two points on the line will have the same color:
|
||||
|
||||
@ -164,7 +164,7 @@
|
||||
\end{tikzpicture}
|
||||
\end{center}
|
||||
|
||||
Our original assumption also implies that the center point is white. \\
|
||||
Our original assumption also implies that the center point is white. \par
|
||||
This, however, creates a line of equidistant white points:
|
||||
|
||||
\begin{center}
|
||||
@ -249,7 +249,7 @@
|
||||
Choose $n + 1$ integers between $1$ and $2n$. Show that you must select two numbers $a$ and $b$ such that $a$ divides $b$.
|
||||
|
||||
\begin{solution}
|
||||
Split the the set $\{1, ..., 2n\}$ into classes defined by each integer's greatest odd divisor. There will be $n$ classes since there are $\frac{k}{2}$ odd numbers between $1$ and $n$. Because we pick $n + 1$ numbers, at least two will come from the same class---they will be divisible. \\
|
||||
Split the the set $\{1, ..., 2n\}$ into classes defined by each integer's greatest odd divisor. There will be $n$ classes since there are $\frac{k}{2}$ odd numbers between $1$ and $n$. Because we pick $n + 1$ numbers, at least two will come from the same class---they will be divisible. \par
|
||||
|
||||
For example, if $n = 5$, our classes are
|
||||
\begin{itemize}
|
||||
@ -286,12 +286,12 @@
|
||||
|
||||
\problem{}
|
||||
\note{Difficulty: Normal}
|
||||
Let $n$ be an odd number. Let $a_1, a_2, ... , a_n$ be a permutation of the numbers $1, 2, ... , n$. \\
|
||||
Let $n$ be an odd number. Let $a_1, a_2, ... , a_n$ be a permutation of the numbers $1, 2, ... , n$. \par
|
||||
Show that $(a_1 - 1) \times (a_2 - 2) \times ... \times (a_n - n)$ is even.
|
||||
|
||||
\begin{solution}
|
||||
If $n$ is odd, there will be $m$ even and $m + 1$ odd numbers between $1$ and $n$. \\
|
||||
Therefore, if we match each $a_n$ with an integer in $[1, ..., n]$, we will have to match at least one odd number with an odd number. \\
|
||||
If $n$ is odd, there will be $m$ even and $m + 1$ odd numbers between $1$ and $n$. \par
|
||||
Therefore, if we match each $a_n$ with an integer in $[1, ..., n]$, we will have to match at least one odd number with an odd number. \par
|
||||
|
||||
The difference of two odd numbers is even, so the product above will have at least one factor of two.
|
||||
\end{solution}
|
||||
@ -349,13 +349,13 @@
|
||||
\end{tikzpicture}
|
||||
\end{center}
|
||||
|
||||
If there exists a sequence of days where the student drinks exactly 100 espressos, we must have at least one ``block'' (in orange, above) of 100 espressos that both begins and ends on a ``clean break'' between days. \\
|
||||
If there exists a sequence of days where the student drinks exactly 100 espressos, we must have at least one ``block'' (in orange, above) of 100 espressos that both begins and ends on a ``clean break'' between days. \par
|
||||
|
||||
There are $499$ ``breaks'' between $500$ espressos. \\
|
||||
In a year, there are $364$ clean breaks. This leaves $499 - 364 = 135$ ``dirty'' breaks. \\
|
||||
We therefore have $135$ places to start a block on a dirty break, and $135$ places to end a block on a dirty break. This gives us a maximum of $270$ dirty blocks. \\
|
||||
There are $499$ ``breaks'' between $500$ espressos. \par
|
||||
In a year, there are $364$ clean breaks. This leaves $499 - 364 = 135$ ``dirty'' breaks. \par
|
||||
We therefore have $135$ places to start a block on a dirty break, and $135$ places to end a block on a dirty break. This gives us a maximum of $270$ dirty blocks. \par
|
||||
|
||||
However, there are $401$ possible blocks, since we can start one at the $1^{\text{st}}, 2^{\text{nd}}, ..., 401^{\text{st}}$ espresso. \\
|
||||
However, there are $401$ possible blocks, since we can start one at the $1^{\text{st}}, 2^{\text{nd}}, ..., 401^{\text{st}}$ espresso. \par
|
||||
|
||||
Out of $401$ blocks, a maximum of $270$ can be dirty. We are therefore guaranteed at least $131$ clean blocks. This completes the problem---each clean block represents a set of consecutive, whole days during which exactly 100 espressos were consumed.
|
||||
|
||||
|
Loading…
x
Reference in New Issue
Block a user